Difference between revisions of "1998 AJHSME Problems/Problem 23"

(Created page with "If the pattern in the diagram continues, what fraction of the interior would be shaded in the eighth triangle? [http://www.artofproblemsolving.com/Forum/viewtopic.php?p=2331778&...")
(No difference)

Revision as of 12:22, 27 June 2011

If the pattern in the diagram continues, what fraction of the interior would be shaded in the eighth triangle?

picture

$\text{(A)}\ \frac{3}{8}\qquad\text{(B)}\ \frac{5}{27}\qquad\text{(C)}\ \frac{7}{16}\qquad\text{(D)}\ \frac{9}{16}\qquad\text{(E)}\ \frac{11}{45}$